LSAT and Law School Admissions Forum

Get expert LSAT preparation and law school admissions advice from PowerScore Test Preparation.

 Administrator
PowerScore Staff
  • PowerScore Staff
  • Posts: 8916
  • Joined: Feb 02, 2011
|
#35628
Complete Question Explanation

Flaw in the Reasoning. The correct answer choice is (A)

This counselor attempts to make a case for harsh criticism: one needs a motive to change, unpleasant criticism can provide such a motive and, the counselor points out, criticism that is harsh happens to be unpleasant. The counselor then jumps to the conclusion that harsh criticism is the only way to bring change:
  • Premise: ..... Change requires motive

    Premise: ..... Unpleasant (and specifically, harsh) criticism provides such a motive

    Conclusion: ..... Therefore harsh criticism must be the only way to bring change.


So, the counselor determines one means of bringing change, and concludes that this must be the only way.

The question stem requires you to find the best description of this flaw; the problem here is that the counselor chooses one sufficient means (harsh criticism) and then concludes, questionably, that it is the only possibility.

Answer choice (A): This is the correct answer choice. As discussed above, the counselor establishes that harsh criticism is a sufficient motive for change, and then concludes that it is the only possibility—in other words, a necessity.

Answer choice (B): The counselor’s conclusion is that criticism must be harsh—if it is to motivate change. This argument does not involve criticism that isn’t intended to bring change, nor does it need to, so the “failure” to address this possibility is not a flaw in the counselor’s argument, and this cannot be the correct answer choice.

Answer choice (C): The counselor doesn’t presume that motivation will always bring change (this would be diagrammed as: motivation :arrow: change), but instead points out that change requires motive (properly diagrammed as: change :arrow: motive). Since this choice is not accurate, it cannot be the one that describes the stimulus’ flaw.

Answer choice (D): The counselor does not confuse the motivation to do something with the motivation to avoid something, so this is not the flaw in the counselor’s reasoning and cannot be the right answer choice.

Answer choice (E): The counselor doesn’t really refute an argument—but instead questions the notion that criticism should be gentle. Further, the counselor doesn’t say that people who think that criticism should be gentle are necessarily wrong—just that they should consider what it takes to change.
 Garrett K
  • Posts: 28
  • Joined: Jul 28, 2014
|
#22293
Hello PowerScore,

I am confused about question 25...

Why is B an incorrect answer? My pre phrase was that other considerations (excluding harsh criticism) may be able to provide a motive. Was my pre phrase just that off? And with answer A, the language just really confuses me. Can you please further explain the wording in answer A?

Thanks,
Garrett
 Emily Haney-Caron
PowerScore Staff
  • PowerScore Staff
  • Posts: 577
  • Joined: Jan 12, 2012
|
#22294
Hi Garrett,

For 25, A is saying that just because something is sufficient to cause something to happen does not mean it HAS be present for that thing to happen - that is, it is possible that something else could also make the thing happen. That matches your pre-phrase - harsh criticism is enough to lead to change, but it is not necessary for change to happen.

B is just completely irrelevant to the question. We don't know anything about the goals of the criticism, and it just doesn't matter for our purposes - all we're concerned with are the consequences.
 LustingFor!L
  • Posts: 80
  • Joined: Aug 27, 2016
|
#35485
I saw this was a flaw question and there was conditional language in the stimulus. I picked A, because I knew it was likely the correct answer and I didn't have time to read all choices. Looking back, I've diagrammed it as the following:

P: change -> motive
P: criticism that is unpleasant -> motive
P: harsh criticism -> unpleasant
P: harsh criticism -> motive

The premises link up to be the following,
harsh criticism -> unpleasant -> motive
harsh criticism -> motive

C: Person criticized to change :arrow: harsh criticism
This cannot be inferred unless you flip flop harsh criticism --> motive to motive --> harsh criticism?

So, A is saying that she made harsh criticism necessary to motive, even though it's sufficient to motive?
 Adam Tyson
PowerScore Staff
  • PowerScore Staff
  • Posts: 5153
  • Joined: Apr 14, 2011
|
#36067
Perfect job, Lusting! Just remember that on test day, all that extra work is not needed, and in fact it gets in the way of quickly and confidently selecting the correct answer and moving on to other, more challenging questions. Your first approach - recognizing that the argument was conditional and selecting the only answer that addressed conditional flaws - was your best approach, tactically speaking. Save the post-game analysis for after the game, as you did here.
 pasu1223
  • Posts: 12
  • Joined: Aug 21, 2017
|
#38942
Hello all,

Quick question, are "required" and "provides" words that introduce conditional or causal logic?

It seems that the conclusion is clearly causal. How do you recommend bridging the gap between the two or working with both forms of reasoning in a single stimulus?

Thanks!

Patrick
 Adam Tyson
PowerScore Staff
  • PowerScore Staff
  • Posts: 5153
  • Joined: Apr 14, 2011
|
#39377
Good question, and good eye, Patrick - this argument has a series of conditional premises but ends with a causal conclusion. Most of the time, when these two types of reasoning get mashed together, causal reasoning takes priority. This is because it is more powerful, and also more prone to flaws.

"Requires" is absolutely a conditional indicator - it means "makes necessary".

"Provides" is a weird one, in my opinion. It's not exactly conditional, because it is somewhat "active" while conditional language is typically more passive, but it's weaker than typical causal language, which more often sounds like the language of blame ("you did that! you are responsible!") I would most often treat "provides" as a causal indicator ("you provided that!"), although handling it conditionally here seems to be okay too. That's part of why I said that all that work was perhaps unnecessary, because one way or another it feels like we are forcing the stimulus into a framework that is more complex and confusing than it needs to be.

The correct answer also mashes up the conditional with the causal in the same way, and that is what should make it attractive. It's worded as a Mistaken Reversal, with the Sufficient condition being treated as a Necessary condition, but it throws in that weak causal "provides" issue. Call it conditional, call it causal, call it whatever you want so long as you recognize what happened and can pick the answer that describes it.

Final note: Most of the time, conditional premises coupled with a causal conclusion will be best approached with a causal analysis. There could be another cause, or the two could be reversed, or the cause can happen without the effect, etc. Even in this case, that should get you the correct answer because answers B through E have nothing causal OR conditional in them, and answer A has both.

Keep at it!

Get the most out of your LSAT Prep Plus subscription.

Analyze and track your performance with our Testing and Analytics Package.